Prove that $4^n-1$ is divided by $3^k$ if and only if $n=3^k-1t$How many powers of 2 are easy to double?Counting an orbit length in $mathbbZ/q^alphamathbbZ$Simple prime no. quesitonSum of all possible remainders when $2^n$, where n is a nonnegative integer, is divided by 1000Assume that 495 divides the integer $overline273x49y5$ where $x,y in 0,1,2…9$. Find $x$ and $y$.Generalization of the fact that $sum_i=1^nfrac 1 i$ is not an integer for all $n>1$.Prove that gcd between two consecutive terms in a sequence is constant (and find it)Equivalence between a limit and Pillai's ConjectureExpression for the highest power of 2 dividing $3^aleft(2b-1right)-1$Using least common multiple to prove there exists a prime between $2x$ and $3x$

How can saying a song's name be a copyright violation?

If human space travel is limited by the G force vulnerability, is there a way to counter G forces?

Doing something right before you need it - expression for this?

Would Slavery Reparations be considered Bills of Attainder and hence Illegal?

Where does SFDX store details about scratch orgs?

How much of data wrangling is a data scientist's job?

Can I ask the recruiters in my resume to put the reason why I am rejected?

What mechanic is there to disable a threat instead of killing it?

Brothers & sisters

Etiquette around loan refinance - decision is going to cost first broker a lot of money

What's the difference between 'rename' and 'mv'?

Stopping power of mountain vs road bike

1960's book about a plague that kills all white people

How to model explosives?

In Romance of the Three Kingdoms why do people still use bamboo sticks when papers are already invented?

How can I prevent hyper evolved versions of regular creatures from wiping out their cousins?

How can I make my BBEG immortal short of making them a Lich or Vampire?

Arrow those variables!

Why "Having chlorophyll without photosynthesis is actually very dangerous" and "like living with a bomb"?

Python: return float 1.0 as int 1 but float 1.5 as float 1.5

Did Shadowfax go to Valinor?

In a Spin are Both Wings Stalled?

SSH "lag" in LAN on some machines, mixed distros

How to say in German "enjoying home comforts"



Prove that $4^n-1$ is divided by $3^k$ if and only if $n=3^k-1t$


How many powers of 2 are easy to double?Counting an orbit length in $mathbbZ/q^alphamathbbZ$Simple prime no. quesitonSum of all possible remainders when $2^n$, where n is a nonnegative integer, is divided by 1000Assume that 495 divides the integer $overline273x49y5$ where $x,y in 0,1,2…9$. Find $x$ and $y$.Generalization of the fact that $sum_i=1^nfrac 1 i$ is not an integer for all $n>1$.Prove that gcd between two consecutive terms in a sequence is constant (and find it)Equivalence between a limit and Pillai's ConjectureExpression for the highest power of 2 dividing $3^aleft(2b-1right)-1$Using least common multiple to prove there exists a prime between $2x$ and $3x$













0












$begingroup$


I am aware that $3$ always divides $2^2n-1$, which is easy to prove. To deal with some automorphism groups, I am trying to generalise this to find the highest power of $3$ that can divide $2^n -1$.



A reasonable conjecture is that $2^2n-1$ is divided by $3^k$ if and only if $n=3^k-1m$. An example would be that $4^3n+t = 1 mod 9$ if and only if $t=0$ (where, of course, $0 leq t leq 5$). This is easily proved by showing $2^6n+t neq 1 mod 9$ when $t neq 0$.



I can't, however, find an easy argument to show this for $4^n - 1$, since checking every value of $t$ separately is not a viable strategy.










share|cite|improve this question









$endgroup$
















    0












    $begingroup$


    I am aware that $3$ always divides $2^2n-1$, which is easy to prove. To deal with some automorphism groups, I am trying to generalise this to find the highest power of $3$ that can divide $2^n -1$.



    A reasonable conjecture is that $2^2n-1$ is divided by $3^k$ if and only if $n=3^k-1m$. An example would be that $4^3n+t = 1 mod 9$ if and only if $t=0$ (where, of course, $0 leq t leq 5$). This is easily proved by showing $2^6n+t neq 1 mod 9$ when $t neq 0$.



    I can't, however, find an easy argument to show this for $4^n - 1$, since checking every value of $t$ separately is not a viable strategy.










    share|cite|improve this question









    $endgroup$














      0












      0








      0


      1



      $begingroup$


      I am aware that $3$ always divides $2^2n-1$, which is easy to prove. To deal with some automorphism groups, I am trying to generalise this to find the highest power of $3$ that can divide $2^n -1$.



      A reasonable conjecture is that $2^2n-1$ is divided by $3^k$ if and only if $n=3^k-1m$. An example would be that $4^3n+t = 1 mod 9$ if and only if $t=0$ (where, of course, $0 leq t leq 5$). This is easily proved by showing $2^6n+t neq 1 mod 9$ when $t neq 0$.



      I can't, however, find an easy argument to show this for $4^n - 1$, since checking every value of $t$ separately is not a viable strategy.










      share|cite|improve this question









      $endgroup$




      I am aware that $3$ always divides $2^2n-1$, which is easy to prove. To deal with some automorphism groups, I am trying to generalise this to find the highest power of $3$ that can divide $2^n -1$.



      A reasonable conjecture is that $2^2n-1$ is divided by $3^k$ if and only if $n=3^k-1m$. An example would be that $4^3n+t = 1 mod 9$ if and only if $t=0$ (where, of course, $0 leq t leq 5$). This is easily proved by showing $2^6n+t neq 1 mod 9$ when $t neq 0$.



      I can't, however, find an easy argument to show this for $4^n - 1$, since checking every value of $t$ separately is not a viable strategy.







      number-theory elementary-number-theory






      share|cite|improve this question













      share|cite|improve this question











      share|cite|improve this question




      share|cite|improve this question










      asked Mar 21 at 13:21









      AnalysisStudent0414AnalysisStudent0414

      4,408928




      4,408928




















          1 Answer
          1






          active

          oldest

          votes


















          2












          $begingroup$

          In general if the multiplicative order of $a$ mod $p^k$ is $r$ (where $p$ is prime and $a > 1$ is not divisible by $p$), then the order of $a$ mod $p^k+1$ divides $r p$. However, in some cases it is just $r$. I want to prove that doesn't happen in the case $p=3$, $a=2$.



          Let $nu_p(x)$ denote the highest exponent $k$ such that $p^k | x$.



          Note that $a^3r - 1 = (a^r-1)(a^2r + a^r + 1)$, so $nu_3(a^3r-1) = nu_3(a^r-1) + nu_3(a^2r + a^r + 1)$. If $a^r equiv 1 mod 3$, $a^2r + a^r + 1 equiv 0 mod 3$. However, since $x^2 + x + 1 equiv 0 bmod 9$ has no solutions, it is not $0 bmod 9$. Thus $nu_3(a^2r+a^r+1) = 1$, and $nu_3(a^3r-1) = nu_3(a^r-1) + 1$. By induction, $nu_3(a^3^k r-1) = nu_3(a^r-1) + k$. Since $nu_3(2^2-1) = nu_3(3) = 1$, we have
          $nu_3(2^2cdot 3^k-1) = k+1$.






          share|cite|improve this answer











          $endgroup$













            Your Answer





            StackExchange.ifUsing("editor", function ()
            return StackExchange.using("mathjaxEditing", function ()
            StackExchange.MarkdownEditor.creationCallbacks.add(function (editor, postfix)
            StackExchange.mathjaxEditing.prepareWmdForMathJax(editor, postfix, [["$", "$"], ["\\(","\\)"]]);
            );
            );
            , "mathjax-editing");

            StackExchange.ready(function()
            var channelOptions =
            tags: "".split(" "),
            id: "69"
            ;
            initTagRenderer("".split(" "), "".split(" "), channelOptions);

            StackExchange.using("externalEditor", function()
            // Have to fire editor after snippets, if snippets enabled
            if (StackExchange.settings.snippets.snippetsEnabled)
            StackExchange.using("snippets", function()
            createEditor();
            );

            else
            createEditor();

            );

            function createEditor()
            StackExchange.prepareEditor(
            heartbeatType: 'answer',
            autoActivateHeartbeat: false,
            convertImagesToLinks: true,
            noModals: true,
            showLowRepImageUploadWarning: true,
            reputationToPostImages: 10,
            bindNavPrevention: true,
            postfix: "",
            imageUploader:
            brandingHtml: "Powered by u003ca class="icon-imgur-white" href="https://imgur.com/"u003eu003c/au003e",
            contentPolicyHtml: "User contributions licensed under u003ca href="https://creativecommons.org/licenses/by-sa/3.0/"u003ecc by-sa 3.0 with attribution requiredu003c/au003e u003ca href="https://stackoverflow.com/legal/content-policy"u003e(content policy)u003c/au003e",
            allowUrls: true
            ,
            noCode: true, onDemand: true,
            discardSelector: ".discard-answer"
            ,immediatelyShowMarkdownHelp:true
            );



            );













            draft saved

            draft discarded


















            StackExchange.ready(
            function ()
            StackExchange.openid.initPostLogin('.new-post-login', 'https%3a%2f%2fmath.stackexchange.com%2fquestions%2f3156820%2fprove-that-4n-1-is-divided-by-3k-if-and-only-if-n-3k-1t%23new-answer', 'question_page');

            );

            Post as a guest















            Required, but never shown

























            1 Answer
            1






            active

            oldest

            votes








            1 Answer
            1






            active

            oldest

            votes









            active

            oldest

            votes






            active

            oldest

            votes









            2












            $begingroup$

            In general if the multiplicative order of $a$ mod $p^k$ is $r$ (where $p$ is prime and $a > 1$ is not divisible by $p$), then the order of $a$ mod $p^k+1$ divides $r p$. However, in some cases it is just $r$. I want to prove that doesn't happen in the case $p=3$, $a=2$.



            Let $nu_p(x)$ denote the highest exponent $k$ such that $p^k | x$.



            Note that $a^3r - 1 = (a^r-1)(a^2r + a^r + 1)$, so $nu_3(a^3r-1) = nu_3(a^r-1) + nu_3(a^2r + a^r + 1)$. If $a^r equiv 1 mod 3$, $a^2r + a^r + 1 equiv 0 mod 3$. However, since $x^2 + x + 1 equiv 0 bmod 9$ has no solutions, it is not $0 bmod 9$. Thus $nu_3(a^2r+a^r+1) = 1$, and $nu_3(a^3r-1) = nu_3(a^r-1) + 1$. By induction, $nu_3(a^3^k r-1) = nu_3(a^r-1) + k$. Since $nu_3(2^2-1) = nu_3(3) = 1$, we have
            $nu_3(2^2cdot 3^k-1) = k+1$.






            share|cite|improve this answer











            $endgroup$

















              2












              $begingroup$

              In general if the multiplicative order of $a$ mod $p^k$ is $r$ (where $p$ is prime and $a > 1$ is not divisible by $p$), then the order of $a$ mod $p^k+1$ divides $r p$. However, in some cases it is just $r$. I want to prove that doesn't happen in the case $p=3$, $a=2$.



              Let $nu_p(x)$ denote the highest exponent $k$ such that $p^k | x$.



              Note that $a^3r - 1 = (a^r-1)(a^2r + a^r + 1)$, so $nu_3(a^3r-1) = nu_3(a^r-1) + nu_3(a^2r + a^r + 1)$. If $a^r equiv 1 mod 3$, $a^2r + a^r + 1 equiv 0 mod 3$. However, since $x^2 + x + 1 equiv 0 bmod 9$ has no solutions, it is not $0 bmod 9$. Thus $nu_3(a^2r+a^r+1) = 1$, and $nu_3(a^3r-1) = nu_3(a^r-1) + 1$. By induction, $nu_3(a^3^k r-1) = nu_3(a^r-1) + k$. Since $nu_3(2^2-1) = nu_3(3) = 1$, we have
              $nu_3(2^2cdot 3^k-1) = k+1$.






              share|cite|improve this answer











              $endgroup$















                2












                2








                2





                $begingroup$

                In general if the multiplicative order of $a$ mod $p^k$ is $r$ (where $p$ is prime and $a > 1$ is not divisible by $p$), then the order of $a$ mod $p^k+1$ divides $r p$. However, in some cases it is just $r$. I want to prove that doesn't happen in the case $p=3$, $a=2$.



                Let $nu_p(x)$ denote the highest exponent $k$ such that $p^k | x$.



                Note that $a^3r - 1 = (a^r-1)(a^2r + a^r + 1)$, so $nu_3(a^3r-1) = nu_3(a^r-1) + nu_3(a^2r + a^r + 1)$. If $a^r equiv 1 mod 3$, $a^2r + a^r + 1 equiv 0 mod 3$. However, since $x^2 + x + 1 equiv 0 bmod 9$ has no solutions, it is not $0 bmod 9$. Thus $nu_3(a^2r+a^r+1) = 1$, and $nu_3(a^3r-1) = nu_3(a^r-1) + 1$. By induction, $nu_3(a^3^k r-1) = nu_3(a^r-1) + k$. Since $nu_3(2^2-1) = nu_3(3) = 1$, we have
                $nu_3(2^2cdot 3^k-1) = k+1$.






                share|cite|improve this answer











                $endgroup$



                In general if the multiplicative order of $a$ mod $p^k$ is $r$ (where $p$ is prime and $a > 1$ is not divisible by $p$), then the order of $a$ mod $p^k+1$ divides $r p$. However, in some cases it is just $r$. I want to prove that doesn't happen in the case $p=3$, $a=2$.



                Let $nu_p(x)$ denote the highest exponent $k$ such that $p^k | x$.



                Note that $a^3r - 1 = (a^r-1)(a^2r + a^r + 1)$, so $nu_3(a^3r-1) = nu_3(a^r-1) + nu_3(a^2r + a^r + 1)$. If $a^r equiv 1 mod 3$, $a^2r + a^r + 1 equiv 0 mod 3$. However, since $x^2 + x + 1 equiv 0 bmod 9$ has no solutions, it is not $0 bmod 9$. Thus $nu_3(a^2r+a^r+1) = 1$, and $nu_3(a^3r-1) = nu_3(a^r-1) + 1$. By induction, $nu_3(a^3^k r-1) = nu_3(a^r-1) + k$. Since $nu_3(2^2-1) = nu_3(3) = 1$, we have
                $nu_3(2^2cdot 3^k-1) = k+1$.







                share|cite|improve this answer














                share|cite|improve this answer



                share|cite|improve this answer








                edited Mar 21 at 18:40

























                answered Mar 21 at 14:19









                Robert IsraelRobert Israel

                330k23219473




                330k23219473



























                    draft saved

                    draft discarded
















































                    Thanks for contributing an answer to Mathematics Stack Exchange!


                    • Please be sure to answer the question. Provide details and share your research!

                    But avoid


                    • Asking for help, clarification, or responding to other answers.

                    • Making statements based on opinion; back them up with references or personal experience.

                    Use MathJax to format equations. MathJax reference.


                    To learn more, see our tips on writing great answers.




                    draft saved


                    draft discarded














                    StackExchange.ready(
                    function ()
                    StackExchange.openid.initPostLogin('.new-post-login', 'https%3a%2f%2fmath.stackexchange.com%2fquestions%2f3156820%2fprove-that-4n-1-is-divided-by-3k-if-and-only-if-n-3k-1t%23new-answer', 'question_page');

                    );

                    Post as a guest















                    Required, but never shown





















































                    Required, but never shown














                    Required, but never shown












                    Required, but never shown







                    Required, but never shown

































                    Required, but never shown














                    Required, but never shown












                    Required, but never shown







                    Required, but never shown







                    Popular posts from this blog

                    Lowndes Grove History Architecture References Navigation menu32°48′6″N 79°57′58″W / 32.80167°N 79.96611°W / 32.80167; -79.9661132°48′6″N 79°57′58″W / 32.80167°N 79.96611°W / 32.80167; -79.9661178002500"National Register Information System"Historic houses of South Carolina"Lowndes Grove""+32° 48' 6.00", −79° 57' 58.00""Lowndes Grove, Charleston County (260 St. Margaret St., Charleston)""Lowndes Grove"The Charleston ExpositionIt Happened in South Carolina"Lowndes Grove (House), Saint Margaret Street & Sixth Avenue, Charleston, Charleston County, SC(Photographs)"Plantations of the Carolina Low Countrye

                    random experiment with two different functions on unit interval Announcing the arrival of Valued Associate #679: Cesar Manara Planned maintenance scheduled April 23, 2019 at 00:00UTC (8:00pm US/Eastern)Random variable and probability space notionsRandom Walk with EdgesFinding functions where the increase over a random interval is Poisson distributedNumber of days until dayCan an observed event in fact be of zero probability?Unit random processmodels of coins and uniform distributionHow to get the number of successes given $n$ trials , probability $P$ and a random variable $X$Absorbing Markov chain in a computer. Is “almost every” turned into always convergence in computer executions?Stopped random walk is not uniformly integrable

                    How should I support this large drywall patch? Planned maintenance scheduled April 23, 2019 at 00:00UTC (8:00pm US/Eastern) Announcing the arrival of Valued Associate #679: Cesar Manara Unicorn Meta Zoo #1: Why another podcast?How do I cover large gaps in drywall?How do I keep drywall around a patch from crumbling?Can I glue a second layer of drywall?How to patch long strip on drywall?Large drywall patch: how to avoid bulging seams?Drywall Mesh Patch vs. Bulge? To remove or not to remove?How to fix this drywall job?Prep drywall before backsplashWhat's the best way to fix this horrible drywall patch job?Drywall patching using 3M Patch Plus Primer